¿Qué estoy malinterpretando en los circuitos eléctricos con respecto a voltaje/corriente/resistencia?

Pensé que entendía la electricidad, pero algunas cosas no encajan, así que espero que alguien pueda decirme qué estoy haciendo mal. Entonces, para mí, el voltaje representa una capacidad/energía potencial de una carga para hacer algún trabajo/transferir energía. La resistencia usa parte de esa energía para calentarse o hacer otra cosa (realmente no importa qué), para reducir el voltaje de la carga. Al dejar caer el voltaje, también reduce la corriente (porque la carga ahora "necesita menos" llegar al otro terminal). Con esta comprensión de la electricidad, traté de entender el siguiente circuito:

ingrese la descripción de la imagen aquí

esquemático

simular este circuito : esquema creado con CircuitLab

La fuente de voltaje es de 3V. La corriente se calcula fácilmente como 1A.

Así va la historia: la fuente le está dando a 1 amperio de corriente una 'capacidad' de 3 voltios. La primera resistencia provoca una caída de voltaje de 2V. Ahora a ese amperio solo le queda 1 voltio de 'capacidad'. La corriente llega a la rama. Se divide por igual, 0.5A va a cada rama. ¡Pero la resistencia de las resistencias en ramas paralelas es de 2 ohmios, y solo me queda 1V por amperio! No solo eso, sino que mi corriente que pasa a través de la resistencia ya no es ni siquiera 1A, ahora es solo 0.5A, lo que significa que solo le quedan 0.5V de 'capacidad'. Entonces, ¿qué está pasando aquí?

Una pregunta secundaria que surge: según las leyes de Kirchhoff, la suma de las caídas de voltaje debe ser igual al voltaje de la fuente. Entonces, al final, el voltaje de una carga es 0 (después de que pasa por la última resistencia). ¿Por qué incluso se mueve entonces, si su 'disposición' para llegar a la otra terminal ahora es 0?

Sugerencia: use el editor de circuitos integrados para crear un circuito claro y bien etiquetado la próxima vez. Es difícil referirse a cada una de sus tres resistencias por separado si no las etiqueta,
"la resistencia de las resistencias en las ramas paralelas es de 2 ohmios" recuerda que n resistencias iguales en paralelo son lo mismo que una sola resistencia en serie de ohmios/n. Lo que tienes allí es lo mismo que una sola resistencia de un ohmio. Súmelo a r3 y obtendrá una resistencia total de 3 ohmios que, a su vez, según la ley de ohmios, dados sus 3 voltios da como resultado exactamente 1A. Inviértalo y obtendrá, del 1A, sus 3 voltios iniciales.
El voltaje (potencial eléctrico) es muy parecido a la altitud sobre el suelo. Pero la altitud por sí sola no puede empujar un objeto, y el voltaje por sí solo no puede empujar un electrón. La 'capacidad' de la que está hablando no es el voltaje de cada partícula cargada en el cable. En cambio, es la caída de voltaje ; el cambio de voltaje medido a lo largo de una resistencia completa. (Una roca que se desliza es impulsada por la pendiente de una colina, y las cargas dentro de los cables son impulsadas por la "pendiente de voltaje". La altitud de una sola ubicación es irrelevante, la "pendiente" es lo que importa).
La clave para ver es que si observa R3 y R1/R2 como una unidad, la misma corriente debe pasar a través de cada uno, pero cada uno puede tener un voltaje diferente a través de ellos. Sin embargo, si observa R1 y R2, cada uno debe tener el mismo voltaje a través de ellos, pero cada uno puede tener una cantidad diferente de corriente a través de ellos. El voltaje se divide entre R3 y R1/R2, pero R1 y R2 tienen el mismo voltaje entre ellos. La corriente se divide entre R1 y R2, pero R3 tiene la misma corriente que pasa a través de él cuando R1 y R2 se han combinado.

Respuestas (4)

Voy a centrarme en tu error:

La fuente le está dando a 1 amperio de corriente una 'capacidad' de 3 voltios.

Uso divertido de los términos, pero está bien. En mis términos, 3 voltios por amperio es lo mismo que decir 3 ohmios.

La primera resistencia provoca una caída de voltaje de 2V.

Sí. Se necesitan 2 V de fuerza motriz para hacer que 1 A de corriente fluya a través de 2 ohmios de resistencia.

Ahora a ese amperio solo le queda 1 voltio de 'capacidad'.

Ahora estás 'bien' pero comenzando a pisar hielo delgado. Pero sí, queda 1 voltio de diferencia de potencial y todo el 1 A de corriente que ha llegado a este punto, también debe salir de este punto. Y para hacerlo, solo debe enfrentar un Ohm restante de resistencia.

La corriente llega a la rama. Se divide por igual, 0.5A va a cada rama.

Sí. La diferencia de potencial restante (1 V) solo puede causar 0,5 A en una resistencia de 2 ohmios. Afortunadamente, hay dos de esas resistencias, por lo que, afortunadamente, toda la corriente entrante puede encontrar una salida para moverse dada la diferencia de potencial restante.

¡Pero la resistencia de las resistencias en ramas paralelas es de 2 ohmios, y solo me queda 1V por amperio!

Aquí es donde de repente saltas las pistas. Un voltio por amperio significa un ohmio. Eso es lo que significa. Y resulta que allí también hay solo un ohmio. Buena cosa.

Permítame volver a escribir su declaración: "Pero la resistencia de las resistencias en ramas paralelas es de 2 ohmios, ¡y solo tengo 1 ohmio!"

No cambié ni un ápice de significado allí. Dice exactamente lo que acabas de escribir. Pero ahora puedes ver que hay un conflicto. Simultáneamente estás diciendo que hay 2 ohmios y 1 ohmio. Y eso no tiene sentido.

El hecho es que hay 1 ohmio entre los dos puntos en consideración, no 2 ohmios. Sí, hay dos caminos galvánicos y cada uno de estos caminos representa 2 ohmios cada uno. Y la corriente solo será de 0,5 A para cada uno de estos dos caminos, como debe ser. Pero el resultado combinado sigue siendo de 1 ohmio y la corriente combinada seguirá siendo de 1 A.

Solo necesitas arreglar tus modelos mentales.

Acerca de:

No solo eso, sino que mi corriente que pasa a través de la resistencia ya no es ni siquiera 1A, ahora es solo 0.5A, lo que significa que solo le quedan 0.5V de 'capacidad'. Entonces, ¿qué está pasando aquí?

Tiene razón en que la corriente se divide, y cada resistencia (en este caso particular) tiene 0.5 A que la atraviesa, pero el voltaje allí no se divide, por V = I R , medio amperio de corriente a través de una resistencia de dos ohmios significa que todavía hay 1 voltio en esas resistencias.

:) no lo hagas, ¡lo arreglaste!

Al caer el voltaje, también reduce la corriente.

no.

La corriente permanece igual: la cantidad total de cargos debe permanecer constante. Pero debido a que cada una de estas cargas ahora "lleva" menos energía, existe una diferencia de voltaje entre los dos extremos de una resistencia.

Está citando la Ley de Kirchhoff, pero eso solo puede funcionar si reconoce que la corriente debe permanecer igual.

Con eso en mente:

Tiene razón, la simetría nos dice que a través de cada una de las resistencias paralelas R1 y R2 de 2 ohmios, debe fluir la mitad de la corriente en el circuito: 0,5 A.

0,5A * 2 ohmios = 1V

Y junto con la caída de 2 V sobre la resistencia R3 de 2 ohmios en serie que da una caída de voltaje de 1 V + 2 V = 3 V, que se alinea muy bien con su fuente de 3 V.

¡Todo está bien!

Ok, cometí un error allí, pero no creo que sea tan relevante para el problema que presenté.
Lo es mucho. Debe sumar las corrientes, pero el voltaje en un nodo debe ser el mismo desde todas las perspectivas.
Oh, tienes razón, gracias. Ahora, si sabes la respuesta a la pregunta secundaria, también sería genial.
¿Cuál podría ser?
ah cierto, eso. Realmente debería hacer esto en una pregunta separada, con un circuito aún más simple (solo fuente de voltaje - resistencia).

Cuando hablamos de voltajes, en realidad estamos hablando de energía. Y lo que pasa con la energía es que no siempre es bueno para decirnos qué le sucede a una sola partícula en cada punto del viaje. Su clase de introducción a la física podría haber usado una montaña rusa para describir esto:

Montaña rusa

No tienes que conocer las fuerzas en la montaña rusa a lo largo de toda la pista. Todo lo que tienes que saber es la diferencia de altura entre el inicio y el final. El voltaje es algo así.

Físicamente, esto es lo que sucede. La batería crea un campo eléctrico entre los terminales. Cuando los cables están conectados, las cargas en los cables y las resistencias se organizan para canalizar el campo a través del circuito. El campo empuja a los electrones y los mueve a través de los cables y las resistencias. La fuerza del campo realiza trabajo sobre los electrones, dándoles energía:

W = F d = q mi d

A medida que los electrones se mueven a través de las resistencias, chocan contra cosas, lo que los ralentiza. En otras palabras, ¡pierden energía!

Trabajar con campos eléctricos y cargas directamente sería muy, muy difícil. Afortunadamente, al igual que con la montaña rusa, podemos usar energía (voltaje) por sí mismo. Y en lugar de mirar electrones individuales, podemos mirar el flujo total de carga (corriente). Con eso en mente, aquí hay algunas cosas que podrían ayudar:

  • El voltaje no es algo que los electrones "tengan". Una diferencia de voltaje representa la fuerza del campo eléctrico a través de parte del circuito.
  • Dos resistencias en paralelo tienen el mismo voltaje. Esta es una propiedad del campo eléctrico: ganas o pierdes la misma cantidad de energía cuando viajas entre dos puntos, sin importar el camino que tomes. (Esto también es cierto con la gravedad, por cierto).

Si desea obtener una visión más detallada de lo que sucede con los campos y las cargas en un circuito, consulte este documento , que utiliza esas ideas para presentar la teoría de circuitos. Si no quieres leer todo, solo mira los diagramas.